Sunday, February 19, 2017

Lit Bits: Feb 19, 2017

From the recent medical literature...

0. Kids with Blunt Head Trauma

A. The PECARN TBI rules do not apply to abusive head trauma.

Magana JN, et al. Acad Emerg Med. 2016 Dec 31 [Epub ahead of print]

The Pediatric Emergency Care Applied Research Network (PECARN) traumatic brain injury (TBI) prediction rules were developed to identify children at very low risk for clinically-important TBIs (ciTBIs), for whom computed tomography (CT) scans can typically be obviated.1 The PECARN prediction rules have been validated in several settings and countries.2-5 The PECARN TBI rules, one developed for children younger than 2 years, and the other for those 2 years and older, rely on accurate patient history and physical examination findings gathered at the time of emergency department (ED) presentation.

B. Which Children with Mild Head Trauma and Intracranial Injury Need the ICU?

Greenberg JK, et al. JAMA Pediatr. 2017 Feb 13 [Epub ahead of print]

Key Points
Question: Can routine clinical and imaging variables predict the need for intensive care unit admission among children with mild traumatic brain injury and intracranial injury?

Findings: This analysis found that the presence of midline shift, depressed skull fracture, epidural hematoma, and lower Glasgow Coma Scale score are associated with the need for intensive care unit admission in children with mild traumatic brain injury and intracranial injury. The negative predictive value of having none of these risk factors was 98.8%.

Meaning: Use of these risk criteria can help guide the need for intensive care unit admission in children with mild traumatic brain injury and intracranial injury.

Abstract
Importance: The appropriate treatment of children with mild traumatic brain injury (mTBI) and intracranial injury (ICI) on computed tomographic imaging remains unclear. Evidence-based risk assessments may improve patient safety and reduce resource use.

Objective: To derive a risk score predicting the need for intensive care unit observation in children with mTBI and ICI.

Design, Setting, and Participants: This retrospective analysis of the prospective Pediatric Emergency Care Applied Research Network (PECARN) head injury cohort study included patients enrolled in 25 North American emergency departments from 2004 to 2006. We included patients younger than 18 years with mTBI (Glasgow Coma Scale [GCS] score, 13-15) and ICI on computed tomography. The data analysis was conducted from May 2015 to October 2016.

Main Outcomes and Measures: The primary outcome was the composite of neurosurgical intervention, intubation for more than 24 hours for TBI, or death from TBI. Multivariate logistic regression was used to predict the outcome. The C statistic was used to quantify discrimination, and model performance was internally validated using 10-fold cross-validation. Based on this modeling, the Children's Intracranial Injury Decision Aid score was created.

Results: Among 15 162 children with GCS 13 to 15 head injuries who received head computed tomographic imaging in the emergency department, 839 (5.5%) had ICI. The median ages of those with and without a composite outcome were 7 and 5 years, respectively. Among those patients with ICI, 8.7% (n = 73) experienced the primary outcome, including 8.3% (n = 70) who had a neurosurgical intervention. The only clinical variable significantly associated with outcome was GCS score (odds ratio [OR], 3.4; 95% CI, 1.5-7.4 for GCS score 13 vs 15). Significant radiologic predictors included midline shift (OR, 6.8; 95% CI, 3.4-13.8), depressed skull fracture (OR, 6.5; 95% CI, 3.7-11.4), and epidural hematoma (OR, 3.4; 95% CI, 1.8-6.2). The model C statistic was 0.84 (95% CI, 0.79-0.88); the 10-fold cross-validated C statistic was 0.83. Based on this modeling, we developed the Children's Intracranial Injury Decision Aid score, which ranged from 0 to 24 points. The negative predictive value of having 0 points (ie, none of these risk factors) was 98.8% (95% CI, 97.3%-99.6%).

Conclusions and Relevance: Lower GCS score, midline shift, depressed skull fracture, and epidural hematoma are key risk factors for needing intensive care unit-level care in children with mTBI and ICI. Based on these results, the Children's Intracranial Injury Decision Aid score is a potentially novel tool to risk stratify this population, thereby aiding management decisions.


1. New Sepsis Guidelines: January 2017

A. Management of Sepsis and Septic Shock: JAMA Clinical Guidelines Synopsis 

Howell MD, et al. JAMA 2017 Jan 19 [Epub ahead of print]

Sepsis results when the body’s response to infection causes life-threatening organ dysfunction. Septic shock is sepsis that results in tissue hypoperfusion, with vasopressor-requiring hypotension and elevated lactate levels.1 Sepsis is a leading cause of death, morbidity, and expense, contributing to one-third to half of deaths of hospitalized patients,2 depending on definitions.3 Management of sepsis is a complicated clinical challenge requiring early recognition and management of infection, hemodynamic issues, and other organ dysfunctions.

The guideline was developed by the SSC, with funding and governance from the SCCM and the ESICM (Table).4 More than 30 additional organizations endorsed the guidelines. Guideline committee members were from numerous specialties and included methods experts and a patient representative. A formal conflict of interest management policy was followed.

The guideline committee used the GRADE method. Population, intervention, control, and outcomes questions were constructed; professional librarians assisted with evidence reviews. Although the 2016 revision of definitions for sepsis were published during the guideline development process,1 studies used for guideline evidence used earlier definitions of sepsis syndromes.

The 2012 sepsis guidelines strongly recommended protocolized resuscitation with quantitative end points (early goal-directed therapy [EGDT]). Recommendations included specific goals for central venous pressure (CVP), MAP, and central venous oxygen saturation and formed the basis of national quality and performance metrics.5

Since the 2012 guideline, substantial evolution has occurred in understanding the value of EGDT. Three key randomized trials enrolled patients presenting to the emergency department who had sepsis with shock or hypoperfusion. In the PROCESS trial (n=1341 patients from 31 US institutions), protocol-based approaches did not reduce 60-day mortality vs usual care (19.5% vs 18.9%; relative risk [RR], 1.04; 95% CI, 0.82-1.31; P=.83).6 The similarly sized UK-based PROMISE7 and the ARISE trial8 from Australia and New Zealand both compared EGDT and usual care at 90 days and again found no difference in mortality (29.5% vs. 29.2%; RR, 1.01; 95% CI, 0.85-1.20; P=.90 and 18.6% vs 18.8%; RR, 0.98; 95% CI, 0.80-1.21; P=.90, respectively). Taken together, these trials suggest that while EGDT is safe, it is not superior to usual, nonprotocolized care. Usual care has also evolved since these trials to include more aggressive fluid resuscitation.9 In response, the 2016 guideline has removed standard EGDT resuscitation targets, instead recommending that sepsis-induced hypoperfusion be treated with at least 30 mL/kg of intravenous crystalloid given in 3 hours or less. The authors note that sparse controlled data support this volume but its delivery allows resuscitation to begin during evaluation. In the absence of the former static EGDT targets (eg, CVP), the guideline emphasizes frequent clinical reassessment and the use of dynamic measures of fluid responsiveness (eg, arterial pulse pressure variation), given evidence that dynamic measures predict fluid responsiveness better than static measures do.

Because infection causes sepsis, managing infection is perhaps the most critical component of sepsis therapy. Mortality increases even with very short delays of antimicrobials. To optimize the risk-benefit profile, the strategy of initial broad-spectrum therapy requires meticulous attention to antimicrobial stewardship, including early appropriate cultures and daily review to reduce or stop antimicrobials. Additionally, anatomic source control (eg, identifying infected central lines, pyelonephritis with ureteral obstruction, intestinal perforation) should occur as soon as is practical.

The PROCESS,6 PROMISE,7 and ARISE8 trials have created substantial uncertainty in how to guide clinicians managing patients with sepsis and septic shock.9 When usual care is equivalent to EGDT, what is a clinician to do? The most significant update to the guideline reflects this shift in evidence: removing most specific EGDT end points and emphasizing frequent reevaluation and patient-specific tailoring of hemodynamic therapy. Even with a change in consensus definitions for sepsis,1 the guideline provides strong recommendations for a number of elements of standardized care, such as antimicrobial therapy, initial fluid volume, blood pressure goals, and vasopressor choice. Reflecting substantial consensus among experts, voting was by 75% of panel members with at least 80% agreement.

The guideline also provides a BPS for hospitals and health systems to develop formal sepsis performance improvement programs, given a suggestion of a mortality benefit. The recent NICE guidelines include tips on patient populations at higher risk of sepsis, clinical presentations, and initial laboratories for diagnosis and risk stratification. Tools such as order sets, checklists, posters, reminder cards, and electronic medical record decision support may assist clinicians in early recognition and appropriate treatment of sepsis.10

Pediatric sepsis guidelines will be published separately, with a specific guideline for ventilation in ARDS expected in 2017…

Selected major recommendations
Managing infection:

Antibiotics: Administer broad-spectrum intravenous antimicrobials for all likely pathogens within 1 hour after sepsis recognition (strong recommendation; moderate quality of evidence [QOE]).

Source control: Obtain anatomic source control as rapidly as is practical (best practice statement [BPS]).

Antibiotic stewardship: Assess patients daily for deescalation of antimicrobials; narrow therapy based on cultures and/or clinical improvement (BPS).

Managing resuscitation:
Fluids: For patients with sepsis-induced hypoperfusion, provide 30 mL/kg of intravenous crystalloid within 3 hours (strong recommendation; low QOE) with additional fluid based on frequent reassessment (BPS), preferentially using dynamic variables to assess fluid responsiveness (weak recommendation; low QOE).

Resuscitation targets: For patients with septic shock requiring vasopressors, target a mean arterial pressure (MAP) of 65 mm Hg (strong recommendation; moderate QOE).

Vasopressors: Use norepinephrine as a first-choice vasopressor (strong recommendation; moderate QOE).

Mechanical ventilation in patients with sepsis-related ARDS: Target a tidal volume of 6 mL/kg of predicted body weight (strong recommendation; high QOE) and a plateau pressure of ≤30 cm H2O (strong recommendation; moderate QOE).


B. Six Myths Promoted by the New Surviving Sepsis Guidelines (via EMCrit)

By Josh Farkas. EMCrit · January 30, 2017

A. Combination Therapy Is Beneficial For Gram-Negative Septic Shock.

B. Norepinephrine Is The Best Vasopressor For All Septic Patients.

C. If One Vasopressor Doesn’t Work, Additional Drugs Should Be Added In A Sequential Fashion.

D. Lactate Is A Measure Of Tissue Perfusion. Normalization Of Lactate Should Be Used As A Resuscitation Target.

E. All Septic Patients Must Receive 30 Ml/Kg Fluid Initially.

F. It’s Helpful To Mandate That Specific Sepsis Therapies Be Given Within A Rigid Time Frame.


2. Four Appy Studies from Acad Emerg Med

A. MRI vs. US as the initial imaging modality for pediatric and young adult patients with suspected appendicitis.

Imler D, et al. Acad Emerg Med. 2017 Feb 16 [Epub ahead of print]

BACKGROUND: While ultrasound (US), given its lack of ionizing radiation is currently the recommended initial imaging study of choice for the diagnosis of appendicitis in pediatric and young adult patients, it does have significant shortcomings. US is time intensive, operator dependent, and results in frequent inconclusive studies, thus necessitating further imaging, and admission for observation or repeat clinical visits. A rapid focused Magnetic Resonance Imaging (MRI) for appendicitis has been shown to have definitive sensitivity and specificity, similar to Computed tomography (CT) but without radiation and offers a potential alternative to US.

OBJECTIVE: In this single-center prospective cohort study, we sought to determine the difference in total length of stay and charges between rapid MRI and US as the initial imaging modality in pediatric and young adult patients presenting to the Emergency Department (ED) with suspected appendicitis. We hypothesized that rapid MRI would be more efficient and cost effective than US as the initial imaging modality in the ED diagnosis of appendicitis.

METHODS: A prospective randomized cohort study of consecutive patients was conducted in patients 2-30 years of age in an academic ED with access to both rapid MRI and US imaging modalities 24/7. Prior to the start of the study, the days of the week were randomized to either rapid MRI or US as the initial imaging modality. Physicians evaluated patients with suspected appendicitis per their usual manner. If the physician decided to obtain radiologic imaging, the pre-determined imaging modality for the day of the week was used. All decisions regarding other diagnostic testing and/or further imaging were left to the physician's discretion. Time intervals (min) between triage, order placement, start of imaging, end of imaging, image result and disposition (discharge vs. admission), as well as total charges (diagnostic testing, imaging and repeat ED visits) were recorded.

RESULTS: Over a 100-day period, 82 patients were imaged to evaluate for appendicitis; 45/82 (55%) of patients were in the US first group; and 37/82 (45%) patients were in the rapid MRI first group. There were no differences in patient demographics or clinical characteristics between the groups and no cases of missed appendicitis in either group. 11/45 (24%) of US first patients had inconclusive studies, resulting in follow-up rapid MRI and 5 return ED visits contrasted with no inconclusive studies or return visits (p less than 0.05) in the rapid MRI group. The rapid MRI compared to US group was associated with longer ED length of stay (mean difference 100 min; 95% CI 35-169) and increased ED charges (mean difference $4,887; 95% CI $1,821 - $8,513).

CONCLUSIONS: In the diagnosis of appendicitis, US first imaging is more time efficient and less costly than rapid MRI despite inconclusive studies after US imaging. Unless the process of obtaining a rapid MRI becomes more efficient and less expensive, US should be the first line imaging modality for appendicitis in patients 2-30 years of age.

B. Diagnostic Value and Effect of Bedside US in Acute Appendicitis in the ED

Gungor F, et al. Acad Emerg Med. 2017 Feb 7 [Epub ahead of print]

OBJECTIVE: Early and accurate diagnosis of acute appendicitis (AA) with ultrasound can minimize the morbidity and mortality of the patients. In this regard, ultrasound can help to the emergency physicians (EPs) in the diagnosing process and clinical decision making for AA. Therefore, we primarily aimed to evaluate the effectiveness of point of care ultrasound (POCUS) in clinical decision making of EPs for the diagnostic evaluation for AA in the emergency department (ED).

METHODS: The study sample consisted of patients aged over 18 years that presented to the ED with an abdominal pain and underwent diagnostic evaluation for AA. All patients were examined initially with POCUS by EPs and then with radiology-performed ultrasound (RADUS) by radiologists. Pre- and post-POCUS median diagnostic certainty values (MDCVs) for AA were determined with visual analogue scale (VAS) scores (0 not present, 100 certainly present) by POCUS performers. Definitive diagnoses were determined by surgery, pathologic evaluation of appendectomy specimens, or clinical follow results. The sensitivity, specificity, positive likelihood ratio (PLR), and negative likelihood ratio (NLR) for POCUS and RADUS together with pre- and post-POCUS VAS scores for MDCVs were compared.

RESULTS: A total of 264 patients were included into the final analysis and 169 (64%) had a diagnosis of AA. The sensitivity, specificity, PLR, and NLR of ultrasound examinations were 92.3% (95% CI; 87.2-95.8%), 95.8% (89.5-98.8%), 21.9 (8.4-57.2) and 0.08 (0.05-0.1) for POCUS, and 76.9% (69.8-83%), 97.8% (84.9-99.7%), 36.4 (9.25-144.3) and 0.24 (0.18-0.31) for RADUS, respectively. Pre-POCUS and post-POCUS VAS scores for MDCVs were 60 (IQR: 50-65) and 95 (IQR: 20-98), respectively (p=0.000).

CONCLUSION: POCUS, when performed in ED for the diagnosis of AA, has high sensitivity and specificity, and had a positive impact on the clinical decision making of EPs. This article is protected by copyright. All rights reserved.

C. Diagnostic Accuracy of Hx, PE, Lab Tests and Point-of-Care-US for Pediatric Acute Appendicitis in the ED: A Systematic Review and Meta-Analysis.

Benabbas R, et al. Acad Emerg Med. 2017 Feb 18 [Epub ahead of print]

BACKGROUND: Acute appendicitis (AA) is the most common surgical emergency in children. Accurate and timely diagnosis is crucial but challenging due to atypical presentations and the inherent difficulty of obtaining a reliable history and physical examination in younger children.

OBJECTIVES: To determine the utility of history, physical exam, laboratory tests, Pediatric Appendicitis Score (PAS) and Emergency Department-Point-of-Care Ultrasound (ED-POCUS) in the diagnosis of AA in ED pediatric patients. We performed a systematic review and meta-analysis and used a test-treatment threshold model to identify diagnostic findings that could rule in/out AA and obviate the need for further imaging studies specifically, CT scan, MRI and Radiology Department Ultrasound (RUS).

METHODS: We searched PUBMED, EMBASE, and SCOPUS up to October 2016 for studies on ED pediatric patients with abdominal pain. Quality Assessment Tool for Diagnostic Accuracy Studies (QUADAS-2) was used to evaluate the quality and applicability of included studies. Positive and negative Likelihood Ratios (LR+ and LR-) for diagnostic modalities were calculated and when appropriate data was pooled using Meta-DiSc. Based on the available literature on the test characteristics of different imaging modalities and applying Pauker-Kassirer method we developed a test-treatment threshold model.

RESULTS: Twenty-one studies were included encompassing 8,605 patients with AA prevalence of 39.2%. Studies had variable quality using the QUADAS-2 tool with most studies at high risk of partial verification bias. We divided studies based on their inclusion criteria into two groups of "undifferentiated abdominal pain" and abdominal pain "suspected of AA". In patients with "undifferentiated abdominal pain" history of "pain migration to RLQ" (LR+ 4.81, 95% CI 4.81-6.44) and presence of "cough/hop pain" in the physical exam (LR+ 7.64, 95% CI 5.94-9.83) were most strongly associated with AA. In patients "suspected of AA" none of the history or laboratory findings were strongly associated with AA. Rovsing's sign was the physical exam finding most strongly associated with AA (LR+ 3.52, 95% CI 2.65-4.68). Among different PAS cutoff points PAS≥ 9 (LR+ 5.26, 95% CI 3.34-8.29) was most associated with AA. None of the history, physical exam, lab tests findings or PAS alone could rule in or rule out AA in patients with "undifferentiated abdominal pain" or those "suspected of AA". Emergency Department Point-of-Care Ultrasound (ED-POCUS) had LR+ 9.24 (95% CI 6.24-13.28) and LR- 0.17 (95% CI 0.09-0.30). Using our test-treatment threshold model, positive ED-POCUS could rule in AA without the use of CT and MRI, but negative ED-POCUS could not rule out AA.

CONCLUSION: Presence of AA is more likely in patients with undifferentiated abdominal pain migrating to the RLQ or when cough/hop pain is present in the physical exam. Once AA is suspected, no single history, physical exam, lab finding or score attained on PAS can eliminate the need for imaging studies. Test characteristics of ED-POCUS are similar to those reported for RUS in literature for diagnosis of AA. In ED patients suspected of AA, a positive ED-POCUS is diagnostic and obviates the need for CT or MRI while negative ED-POCUS is not enough to rule out AA.

D. Association of Hospital Resources and Imaging Choice for Appendicitis in Pediatric EDs.

Fullerton K, et al. Acad Emerg Med. 2016 Dec 31 [Epub ahead of print]

OBJECTIVE: Abdominal pain and concern for appendicitis are common chief complaints in patients presenting to the pediatric emergency department (PED)1 . Although many professional organizations recommend decreasing use of CT and choosing ultrasound as first line imaging for pediatric appendicitis, significant variability persists in imaging utilization2,3,4 . This study investigated practice variation across children's hospitals in the diagnostic imaging evaluation of appendicitis and determined hospital-level characteristics associated with the likelihood of ultrasound as the first imaging modality.

METHODS: This was a multicenter (seven children's hospitals) retrospective investigation. Data from chart review of 160 consecutive patients aged 3-18 years diagnosed with appendicitis from each site was compared with a survey of site medical directors regarding hospital resource availability, usual practices, and departmental level demographics.

RESULTS: In the diagnostic evaluation of 1090 children with appendicitis, CT scan was performed first for 22.4% of patients, with a range across PEDs of 3.1-83.8%. Ultrasound was performed for 54.0% of patients with a range of 2.5-96.9%. The only hospital level factor significantly associated with ultrasound as the first imaging modality was 24-hour availability of in-house ultrasound (OR 29.2, 95% CI 1.2-691.8).

CONCLUSION: Across children's hospitals, significant practice variation exists regarding diagnostic imaging in the evaluation of patients with appendicitis. Variation in hospital level resources may impact the diagnostic evaluation of patients with appendicitis. Availability of 24-hour in house ultrasound significantly increases the likelihood of ultrasound as first imaging and decreases CT scans. Hospitals aiming to increase the use of ultrasound should consider adding 24-hour in house coverage.

3. Simple Acute Low Back Pain: “Naproxen and Heat are Hard to Beat”

A. Diazepam Is No Better Than Placebo When Added to Naproxen for Acute Low Back Pain.

Friedman BW, et al. Ann Emerg Med. 2017 Jan 19 [Epub ahead of print]

STUDY OBJECTIVE: Low back pain causes more than 2.5 million visits to US emergency departments (EDs) annually. Low back pain patients are often treated with nonsteroidal anti-inflammatory drugs and benzodiazepines. The former is an evidence-based intervention, whereas the efficacy of the latter has not been established. We compare pain and functional outcomes 1 week and 3 months after ED discharge among patients randomized to a 1-week course of naproxen+diazepam versus naproxen+placebo.

METHODS: This was a randomized, double-blind, comparative efficacy clinical trial conducted in an urban health care system. Patients presenting with acute, nontraumatic, nonradicular low back pain of no more than a duration of 2 weeks were eligible for enrollment immediately before discharge from an ED if they had a score greater than 5 on the Roland-Morris Disability Questionnaire, a validated 24-item inventory of functional impairment caused by low back pain. Higher scores on the questionnaire indicate greater functional disability. The primary outcome in the trial was improvement in the score between ED discharge and 1 week later. Secondary outcomes included pain intensity 1 week and 3 months after ED discharge, as measured on a 4-point descriptive scale (severe, moderate, mild, and none). All patients were given 20 tablets of naproxen 500 mg, to be taken twice a day as needed for low back pain. Additionally, patients were randomized to receive either 28 tablets of diazepam 5 mg or identical placebo, to be received as 1 or 2 tablets every 12 hours as needed for low back pain. All patients received a standardized 10-minute low back pain educational session before discharge. Using a between-group mean difference of 5 Roland-Morris Disability Questionnaire points, a previously validated threshold for clinical significance, we calculated the need for at least 100 patients with primary outcome data.

RESULTS: Enrollment began in June 2015 and continued for 9 months. Five hundred forty-five patients were screened for eligibility. One hundred fourteen patients met selection criteria and were randomized. Baseline demographic characteristics were not substantially different between the 2 groups. One hundred twelve patients (98%) provided 1-week outcome data. The mean Roland-Morris Disability Questionnaire score of patients randomized to naproxen+diazepam improved by 11 (95% confidence interval [CI] 9 to 13), as did the mean score of patients randomized to naproxen+placebo (11; 95% CI 8 to 13). At 1-week follow-up, 18 of 57 diazepam patients (32%; 95% CI 21% to 45%) reported moderate or severe low back pain versus 12 of 55 placebo patients (22%; 95% CI 13% to 35%). At 3-month follow-up, 6 of 50 diazepam patients (12%; 95% CI 5% to 24%) reported moderate or severe low back pain versus 5 of 53 placebo patients (9%; 95% CI 4% to 21%). Adverse events were reported by 12 of 57 diazepam patients (21%; 95% CI 12% to 33%) and 8 of 55 placebo patients (15%; 95% CI 7% to 26%).

CONCLUSION: Among ED patients with acute, nontraumatic, nonradicular low back pain, naproxen+diazepam did not improve functional outcomes or pain compared with naproxen+placebo 1 week and 3 months after ED discharge.


B. Neither Flexeril nor Oxycodone prn Improved Naproxen for LBP

Friedman BW, et al. Naproxen with Cyclobenzaprine, Oxycodone/Acetaminophen, or Placebo for Treating Acute Low Back Pain: A RCT. JAMA. 2015 Oct 20;314(15):1572-80.

IMPORTANCE: Low back pain (LBP) is responsible for more than 2.5 million visits to US emergency departments (EDs) annually. These patients are usually treated with nonsteroidal anti-inflammatory drugs, acetaminophen, opioids, or skeletal muscle relaxants, often in combination.

OBJECTIVE: To compare functional outcomes and pain at 1 week and 3 months after an ED visit for acute LBP among patients randomized to a 10-day course of (1) naproxen + placebo; (2) naproxen + cyclobenzaprine; or (3) naproxen + oxycodone/acetaminophen.

DESIGN, SETTING, AND PARTICIPANTS: This randomized, double-blind, 3-group study was conducted at one urban ED in the Bronx, New York City. Patients who presented with nontraumatic, nonradicular LBP of 2 weeks' duration or less were eligible for enrollment upon ED discharge if they had a score greater than 5 on the Roland-Morris Disability Questionnaire (RMDQ). The RMDQ is a 24-item questionnaire commonly used to measure LBP and related functional impairment on which 0 indicates no functional impairment and 24 indicates maximum impairment. Beginning in April 2012, a total of 2588 patients were approached for enrollment. Of the 323 deemed eligible for participation, 107 were randomized to receive placebo and 108 each to cyclobenzaprine and to oxycodone/acetaminophen. Follow-up was completed in December 2014.

INTERVENTIONS: All participants were given 20 tablets of naproxen, 500 mg, to be taken twice a day. They were randomized to receive either 60 tablets of placebo; cyclobenzaprine, 5 mg; or oxycodone, 5 mg/acetaminophen, 325 mg. Participants were instructed to take 1 or 2 of these tablets every 8 hours, as needed for LBP. They also received a standardized 10-minute LBP educational session prior to discharge.

MAIN OUTCOMES AND MEASURES: The primary outcome was improvement in RMDQ between ED discharge and 1 week later.

RESULTS: Demographic characteristics were comparable among the 3 groups. At baseline, median RMDQ score in the placebo group was 20 (interquartile range [IQR],17-21), in the cyclobenzaprine group 19 (IQR,17-21), and in the oxycodone/acetaminophen group 20 (IQR,17-22). At 1-week follow-up, the mean RMDQ improvement was 9.8 in the placebo group, 10.1 in the cyclobenzaprine group, and 11.1 in the oxycodone/acetaminophen group. Between-group difference in mean RMDQ improvement for cyclobenzaprine vs placebo was 0.3 (98.3% CI, -2.6 to 3.2; P = .77), for oxycodone/acetaminophen vs placebo, 1.3 (98.3% CI, -1.5 to 4.1; P = .28), and for oxycodone/acetaminophen vs cyclobenzaprine, 0.9 (98.3% CI, -2.1 to 3.9; P = .45).

CONCLUSIONS AND RELEVANCE: Among patients with acute, nontraumatic, nonradicular LBP presenting to the ED, adding cyclobenzaprine or oxycodone/acetaminophen to naproxen alone did not improve functional outcomes or pain at 1-week follow-up. These findings do not support use of these additional medications in this setting.


C. Complementary therapies in addition to medication for patients with nonchronic, nonradicular LBP: a systematic review.

Rothberg S, et al. Am J Emerg Med. 2017 Jan;35(1):55-61.

BACKGROUND: A total of 2.7 million patients present to US emergency departments annually for management of low back pain (LBP). Despite optimal medical therapy, more than 50% remain functionally impaired 3 months later. We performed a systematic review to address the following question: Among patients with nonchronic LBP, does spinal manipulation, massage, exercise, or yoga, when combined with standard medical therapy, improve pain and functional outcomes more than standard medical therapy alone?

METHODS: We used published searches to identify relevant studies, supplemented with our own updated search. Studies were culled from the Cochrane Register of Controlled Trials, Medline, EMBASE, CINAHL, and the Index to Chiropractic Literature. Our goal was to identify randomized studies that included patients with nonradicular LBP of less than 12 weeks' duration that compared the complementary therapy to usual care, sham therapy, or interventions known not to be efficacious, while providing all patients with standard analgesics. The outcomes of interest were improvement in pain scores or measures of functionality.

RESULTS: We identified 2 randomized controlled trials in which chiropractic manipulation + medical therapy failed to show benefit vs medical therapy alone. We identified 4 randomized controlled trials in which exercise therapy + medical therapy failed to show benefit vs medical therapy alone. We did not identify any eligible studies of yoga or massage therapy.

CONCLUSIONS: In conclusion, for patients with nonchronic, nonradicular LBP, available evidence does not support the use of spinal manipulation or exercise therapy in addition to standard medical therapy. There is insufficient evidence to determine if yoga or massage is beneficial.

D. Predicting three-month functional outcomes after an ED visit for acute LBP.

Friedman BW, et al. Am J Emerg Med. 2017 Feb;35(2):299-305.

BACKGROUND: Nearly 30% of patients who present to an ED with acute, new onset, low back pain (LBP) report LBP-related functional impairment three months later. These patients are at risk of chronic LBP, a highly debilitating condition. It has been reported previously that functional impairment, depression, and psychosomatic symptomatology at the index visit are associated with poor LBP outcomes. We wished to replicate those findings in a cohort of ED patients, and also to determine if clinical features present at one week follow-up could predict three-month outcomes in individual patients.

METHODS: This was a planned analysis of data from a randomized comparative effectiveness study of three analgesic combinations conducted in one ED. Patients were followed by telephone one week and three months post-ED visit. The primary outcome was a three-month Roland-Morris Disability Questionnaire (RMDQ) score over 0, indicating the presence of LBP-related functional impairment. At the index visit, we measured functional impairment (using the RMDQ), depressive symptomatology (using the Patient Health Questionnaire depression module), and psychosomatic features (using the 5-item Cassandra scale). At the one-week follow-up, we ascertained the presence or absence of LBP. We built a logistic regression model in which all the predictors were entered and retained in the model, in addition to socio-demographic variables and dummy variables controlling for investigational medication. Results are reported as adjusted odds ratios (adjOR) with 95% CI. To determine if statistically significant associations could be used to predict three-month outcomes in individual patients, we then calculated positive and negative likelihood ratios [LR(+) and LR(-)] with 95% CI for those independent variables associated with the primary outcome.

RESULTS: Of 295 patients who completed the study, 14 (5%) were depressed and 18 (6%) reported psychosomatic symptoms. The median index visit RMDQ score was 19 (IQR: 17, 21) indicating substantial functional impairment. One week after the ED visit, 193 (65%) patients reported presence of LBP. 294 patients provided a three-month RMDQ score, 88 of whom (30%, 95% CI: 25, 35%) reported a score above 0. Neither depression (adjOR 0.7 [95% CI 0.2, 3.1]), psychosomatic symptomatology (adjOR 0.5 [95% CI 0.1, 2.0]), nor index visit functional impairment (adjOR 1.0 [95% CI 1.0, 1.1]) were associated with three-month outcome. Pain at one week was strongly and independently associated with the three-month outcome when examined at the group level (adjOR 4.0 [95% CI 2.1, 7.7]). However, likelihood ratios for pain or its absence at one-week were insufficiently robust to be clinically useful in predicting three-month outcomes in individual patients (LR+: 1.4 [95% CI: 1.3, 1.7]; LR-: 0.4 [95% CI: 0.2, 0.6]).

CONCLUSIONS: In spite of a strong association at the group level between presence of LBP at one week and functional impairment at three months, when used to predict outcomes in individual patients, presence of pain failed to discriminate with clinically meaningful utility between acute LBP patients destined to have a favorable versus unfavorable three-month outcome.

E. Noninvasive Treatments for Acute, Subacute, and Chronic Low Back Pain: A Clinical Practice Guideline from the American College of Physicians

Qaseem A, et al. Ann Intern Med. 2017 Feb 14 [Epub ahead of print]

Description: The American College of Physicians (ACP) developed this guideline to present the evidence and provide clinical recommendations on noninvasive treatment of low back pain.

Methods: Using the ACP grading system, the committee based these recommendations on a systematic review of randomized, controlled trials and systematic reviews published through April 2015 on noninvasive pharmacologic and nonpharmacologic treatments for low back pain. Updated searches were performed through November 2016. Clinical outcomes evaluated included reduction or elimination of low back pain, improvement in back-specific and overall function, improvement in health-related quality of life, reduction in work disability and return to work, global improvement, number of back pain episodes or time between episodes, patient satisfaction, and adverse effects.

Target Audience and Patient Population:
The target audience for this guideline includes all clinicians, and the target patient population includes adults with acute, subacute, or chronic low back pain.

Recommendation 1: Given that most patients with acute or subacute low back pain improve over time regardless of treatment, clinicians and patients should select nonpharmacologic treatment with superficial heat (moderate-quality evidence), massage, acupuncture, or spinal manipulation (low-quality evidence). If pharmacologic treatment is desired, clinicians and patients should select nonsteroidal anti-inflammatory drugs or skeletal muscle relaxants (moderate-quality evidence). (Grade: strong recommendation)

Recommendation 2: For patients with chronic low back pain, clinicians and patients should initially select nonpharmacologic treatment with exercise, multidisciplinary rehabilitation, acupuncture, mindfulness-based stress reduction (moderate-quality evidence), tai chi, yoga, motor control exercise, progressive relaxation, electromyography biofeedback, low-level laser therapy, operant therapy, cognitive behavioral therapy, or spinal manipulation (low-quality evidence). (Grade: strong recommendation)

Recommendation 3: In patients with chronic low back pain who have had an inadequate response to nonpharmacologic therapy, clinicians and patients should consider pharmacologic treatment with nonsteroidal anti-inflammatory drugs as first-line therapy, or tramadol or duloxetine as second-line therapy. Clinicians should only consider opioids as an option in patients who have failed the aforementioned treatments and only if the potential benefits outweigh the risks for individual patients and after a discussion of known risks and realistic benefits with patients. (Grade: weak recommendation, moderate-quality evidence)


4. Early death after discharge from EDs: analysis of national US insurance claims data

Obermeyer Z, et al. BMJ. 2017 Feb 1;356:j239.

OBJECTIVE: To measure incidence of early death after discharge from emergency departments, and explore potential sources of variation in risk by measurable aspects of hospitals and patients.

DESIGN: Retrospective cohort study.

SETTING:  Claims data from the US Medicare program, covering visits to an emergency department, 2007-12.

PARTICIPANTS:  Nationally representative 20% sample of Medicare fee for service beneficiaries. As the focus was on generally healthy people living in the community, patients in nursing facilities, aged ≥90, receiving palliative or hospice care, or with a diagnosis of a life limiting illnesses, either during emergency department visits (for example, myocardial infarction) or in the year before (for example, malignancy) were excluded.

MAIN OUTCOME MEASURE:  Death within seven days after discharge from the emergency department, excluding patients transferred or admitted as inpatients.

RESULTS:  Among discharged patients, 0.12% (12 375/10 093 678, in the 20% sample over 2007-12) died within seven days, or 10 093 per year nationally. Mean age at death was 69. Leading causes of death on death certificates were atherosclerotic heart disease (13.6%), myocardial infarction (10.3%), and chronic obstructive pulmonary disease (9.6%). Some 2.3% died of narcotic overdose, largely after visits for musculoskeletal problems. Hospitals in the lowest fifth of rates of inpatient admission from the emergency department had the highest rates of early death (0.27%)-3.4 times higher than hospitals in the highest fifth (0.08%)-despite the fact that hospitals with low admission rates served healthier populations, as measured by overall seven day mortality among all comers to the emergency department. Small increases in admission rate were linked to large decreases in risk. In multivariate analysis, emergency departments that saw higher volumes of patients (odds ratio 0.84, 95% confidence interval 0.81 to 0.86) and those with higher charges for visits (0.75, 0.74 to 0.77) had significantly fewer deaths. Certain diagnoses were more common among early deaths compared with other emergency department visits: altered mental status (risk ratio 4.4, 95% confidence interval 3.8 to 5.1), dyspnea (3.1, 2.9 to 3.4), and malaise/fatigue (3.0, 2.9 to 3.7).

CONCLUSIONS:  Every year, a substantial number of Medicare beneficiaries die soon after discharge from emergency departments, despite no diagnosis of a life limiting illnesses recorded in their claims. Further research is needed to explore whether these deaths were preventable.


5. Two Studies on Nursemaid's Elbow

A. Effectiveness of reduction maneuvers in the treatment of nursemaid's elbow: A systematic review and meta-analysis shows hyperpronation is best (hands down--get it?)

Bexkens R, et al. Am J Emerg Med. 2016 Nov 2 [Epub ahead of print].

BACKGROUND/AIM: Nursemaid's elbow usually occurs in young children when longitudinal traction is placed on the arm. Several manipulative maneuvers have been described, although, the most effective treatment technique is yet unclear. The aim of this systematic review and meta-analysis was to compare the two most commonly performed maneuvers (supination-flexion and hyperpronation) in the treatment of nursemaid's elbow.

METHODS: A literature search was performed in PubMed, Embase, and Cochrane databases to identify randomized controlled trials comparing supination-flexion and hyperpronation. Data were extracted and pooled independently by two authors. Methodological quality assessment of included studies was performed. Meta-analysis was performed using a fixed-effect model in case of homogeneity across studies, and using a random-effect model in case of heterogeneity. Heterogeneity was calculated with the χ2 test and inconsistency in study effects across trials was quantified by I2 values.

RESULTS: Seven randomized trials, including 701 patients (62% female), were included. A total of 350 patients were treated with the hyperpronation maneuver versus 351 patients who underwent the supination-flexion maneuver. Meta-analysis showed that hyperpronation was more effective than supination-flexion (risk ratio, 0.34; 95% confidence interval, 0.23 to 0.49; I2, 35%). The absolute risk difference between maneuvers was 26% in favor of hyperpronation, resulting in a number needed to treat of 4 patients. Trials lacked blinding of assessors and universal pain measures.

CONCLUSIONS: Hyperpronation was more effective in terms of success rate and seems to be less painful compared to the supination-flexion maneuver in children with nursemaid's elbow.

B. Radial Head Subluxation: Factors Associated with Its Recurrence and Radiographic Evaluation in a Tertiary Pediatric ED

Wong K, et al. J Emerg Med. 2016 Dec;51(6):621-627

BACKGROUND: Radial head subluxation (RHS) is a common complaint seen in the pediatric emergency department in children ages 6 months to 4 years. Classically, injury occurs due to axial traction on the arm, but this mechanism is not universal. Some patients will have recurrent RHS; some may undergo x-ray (XR) evaluation for alternative diagnosis.

OBJECTIVES: To determine factors associated with recurrences and radiographic evaluations in RHS.

METHODS: A retrospective study with inclusion criteria: under 10 years of age with discharge diagnosis "nursemaid," "radial head," or "subluxation." We examined factors associated with RHS recurrences, circumstances when radiographic evaluations performed, physician's training background (pediatric vs. general emergency medicine), mechanisms of injury, and demographic factors including age, gender, and arm involved.

RESULTS: In 246 visits, median age was 27 months (interquartile range 16.1), with females comprising 55.7% (n = 137), and left-sided predominance (52%, n = 130). Mechanisms of injury were classified as "pull" (65.9%, n = 162), "non-pull" (15.9%, n = 39), and "unknown" (18.3%, n = 45). Eighteen patients with recurring RHS were more likely to be male (p = 0.008). In 61 visits where radiography was performed, patients were older (p = 0.03), with a higher frequency seen in non-pull and unknown mechanism (p = 0.0001). No significant difference was found in frequency of radiographs obtained in regard to physician training (p = 0.4660).

CONCLUSION: RHS can result from a myriad of mechanisms. We found that recurrence was more likely in male patients. Factors associated with radiographic evaluation included atypical mechanism, older age, and unclear history, regardless of physician training background.

6. Clinical Policy: Critical Issues in the Initial Evaluation and Management of Patients Presenting to the ED in Early Pregnancy

Hahn SA, et al. Ann Emerg Med 2017;69(2):241–250.e20.

This clinical policy from the American College of Emergency Physicians is an update of the 2012 Clinical Policy: Critical Issues in the Initial Evaluation and Management of Patients Presenting to the Emergency Department in Early Pregnancy.1

A writing subcommittee reviewed the literature to derive evidence-based recommendations to help clinicians answer the following critical questions: (1) Should the emergency physician obtain a pelvic ultrasound in a clinically stable pregnant patient who presents to the emergency department with abdominal pain and/or vaginal bleeding and a β-human chorionic gonadotropin (β-hCG) level below a discriminatory threshold? (2) In patients who have an indeterminate transvaginal ultrasound result, what is the diagnostic utility of β-hCG for predicting possible ectopic pregnancy?

Emergency physicians frequently evaluate and manage patients with abdominal pain and/or vaginal bleeding in the first trimester of pregnancy (also referred to here as “early pregnancy”). Their primary concern in this group of patients is to identify ectopic pregnancy. The prevalence of ectopic pregnancy in symptomatic emergency department (ED) patients is as high as 13% in some series, which is much higher than the incidence in the general population.2, 3

Ultrasound is part of the usual workup for patients with symptomatic early pregnancy. A meta-analysis4 and systematic review5 both found that bedside ultrasound performed by emergency physicians can be used as a screening tool for ectopic pregnancy; however, a review of the evidence supporting this practice is beyond the scope of this policy. The term bedside ultrasound is used here to refer to pelvic ultrasounds that are performed in the ED by the emergency clinician, rather than in the radiology department. In this clinical policy, the term pelvic ultrasound implies the use of a transvaginal approach unless transabdominal images have identified an intrauterine pregnancy. According to the 2014 American College of Emergency Physicians (ACEP) policy statement “Emergency Ultrasound Imaging Criteria Compendium,” the primary indication for bedside ultrasound of the pelvis is to evaluate for the presence of intrauterine pregnancy, thus minimizing the likelihood of an ectopic pregnancy when modifying factors such as infertility treatment (putting patients at risk of heterotopic pregnancy) are not present.6 The multidisciplinary association the American Institute of Ultrasound in Medicine (AIUM) further specifies that the definitive diagnosis of an intrauterine pregnancy be based on visualizing an intrauterine gestational sac containing a yolk sac or embryo-fetus with cardiac activity.7 A bedside ultrasonographer may or may not visualize the adnexa. A comprehensive ultrasound, in contrast, is usually performed in a radiology department and is expected to include views of the uterus, adnexa, and cul-de-sac. Studies using either or both categories of ultrasound were reviewed and this distinction is highlighted in the text and Evidentiary Table.

Ultrasound has facilitated the evaluation of complications of early pregnancy; however, diagnostic algorithms still vary considerably among providers and institutions. Algorithms guiding the evaluation of abdominal pain or vaginal bleeding in early pregnancy generally incorporate the results of quantitative serum β-human chorionic gonadotropin (β-hCG) measurements and pelvic ultrasonography. Many algorithms apply the principle of the discriminatory threshold that historically has been defined as the level at which the sensitivity of ultrasound is thought to approach 100% for the detection of intrauterine pregnancy for the presumptive diagnosis of ectopic pregnancy if an intrauterine pregnancy is not visualized when the β-hCG is above that defined cutoff. This threshold depends on the ultrasound criteria used to define an intrauterine pregnancy and is institution, operator, and patient dependent, but is commonly reported as ranging from 1,000 to 2,000 mIU/mL for transvaginal sonography performed in the radiology department.8, 9 Although the traditionally defined discriminatory threshold has been widely accepted, its applicability to ED practice is not as well established, and the concept itself has been called into question.10, 11 For these reasons, this policy refers to the term “discriminatory threshold” where necessary but does not endorse the concept or refer to any specific β-hCG cutoff level.

The first critical question deals with the diagnostic and management variability that occurs when the clinician obtains a β-hCG result, and it is below a commonly defined discriminatory threshold. Some clinicians may not perform an ultrasound for these patients because of incorrect assumptions (eg, ectopic pregnancy is unlikely because the β-hCG level is low, because of a misunderstanding that the risk of rupture is low in this subgroup). However, it is well documented that ectopic pregnancies can present at almost any β-hCG level, high or low,8 and rupture has been documented at very low β-hCG levels.8, 12 In addition, ultrasound determination of pregnancy location for symptomatic patients has been designated as a Centers for Medicare & Medicaid Services (CMS) Core Quality Measure, with few exclusions such as lack of ultrasound availability. A β-hCG level is not part of the inclusion or exclusion criteria for this CMS core measure.13

The emergency physician is faced with another diagnostic and management question when an ultrasound result is described as indeterminate, “nondiagnostic,” or a “pregnancy of unknown location.” The second critical question examines this subgroup of patients with indeterminate ultrasound results and addresses whether the initial β-hCG level can help risk-stratify these patients….


7. Chest Pain Studies

A. Finally! High-sensitive Trops approved by FDA for use in the U.S.

Breakthrough development for Americans with suspected heart attack - Next generation Troponin T test from Roche cleared by FDA

INDIANAPOLIS, Jan. 19, 2017 /PRNewswire/ -- Roche (SIX: RO, ROG; OTCQX: RHHBY) announced that it has received the 510(k) clearance for its Elecsys Troponin T Gen 5 STAT (TnT Gen 5 STAT) blood test for patients with a suspected heart attack. With this clearance, Roche is the first IVD company in the U.S. to provide the next generation Troponin testing for patients as an aid in the diagnosis of myocardial infarction, enabling clinicians to more accurately identify patients experiencing a heart attack.

Every minute counts
Every 43 seconds someone in the U. S. has a heart attack, or acute myocardial infarction (AMI), which occurs when the blood supply to an area of the heart is interrupted. The longer time the heart is without proper blood supply, the greater the damage.  Troponin, a specific marker of cardiac cell death, is released into the blood stream when cardiac cells are being damaged.

Patients with chest pain and other symptoms suggestive of AMI account for approximately 8 million of all emergency room consultations in the U.S., but only a fraction of them (5-20%) are actually having an AMI. Hence, fast and accurate diagnosis of AMI requires sensitive diagnostic tests that can detect early troponin release, allowing healthcare providers to make confident clinical decisions for their patients and appropriately manage hospital resources. In a heart attack, early diagnosis and initiation of treatment can reduce the amount of cardiac cell death thus potentially saving and improving quality of lives. This next generation Troponin T test from Roche is able to provide accuracy at lower levels of troponin to aid in correctly identifying patients having an AMI.

Moreover, this next generation of TnT test from Roche has been available in the rest of the world for the past seven years during which time it has been rapidly adopted by clinicians and its clinical diagnostic utility is supported by more than 600 peer-reviewed publications.

"As an emergency doctor whose job is to make decisions on patients with chest pain, my ability to do this safely and accurately is driven by the sensitivity of the troponin assay. FDA clearance of this new Roche TnT assay is easily the best news in the last decade for emergency medicine patients presenting with chest pain." Frank Peacock MD, FACEP – ED Physician, Baylor Medical Center Houston – Houston, TX

B. Observation Status or Inpatient Admission: Impact of Patient Disposition on Outcomes and Utilization among ED Patients w/ CP

Bellolio MF, et al. Acad Emerg Med. 2017 Feb;24(2):152-160.

OBJECTIVES: to compare healthcare utilization including coronary angiography, percutaneous coronary intervention (PCI), rehospitalization, and rate of subsequent acute myocardial infarction (AMI) within 30 days, among patients presenting to the emergency department (ED) with chest pain admitted as short-term inpatient (≤2 days) versus observation (in-ED observation units combined with in-hospital observation).

METHODS: We identified adults diagnosed with acute chest pain in the ED from 2010 to 2014 using administrative claims from privately insured and Medicare Advantage. Patients having AMI during the index visit were excluded. One-to-one propensity-score matching and logistic regression were used. Odds ratios (ORs) with 95% confidence intervals (CIs) were reported.

RESULTS: A total of 774,017 chest pain visits were included. After matching, healthcare utilization was lower among observation versus short inpatient, with 10.9% versus 24.4% (OR = 0.38, 95% CI = 0.36 to 0.39) undergoing cardiac catheterization and 1.8% versus 7.6% (OR = 0.23, 95% CI = 0.21 to 0.24) having PCI. The incidence of subsequent AMI within the following 30 days was similar in patients admitted as observation versus short inpatient (0.23% vs. 0.21%; OR = 1.09, 95% CI = 0.84 to 1.42).

CONCLUSIONS: There were higher rates of cardiac catheterization and PCI among those admitted as a short inpatient compared to observation, while the incidence of subsequent AMI within 30 days was similar.

8. The Conversation Placebo

Ofrijan D. NY Times, 2017 Jan 19

In my daily work as a primary care internist, I see no letup from pain. Every single patient, it seems, has an aching shoulder or a bum knee or a painful back. “Our bodies evolved to live about 40 years,” I always explain, “and then be finished off by a mammoth or a microbe.” Thanks to a century of staggering medical progress, we now live past 80, but evolution hasn’t caught up; the cartilage in our joints still wears down in our 40s, and we are more obese and more sedentary than we used to be, which doesn’t help.

So it’s no surprise that chronic arthritis and back pain are the second and third most common non-acute reasons that people go to the doctor and that pain costs America up to $635 billion annually. The pain remedies developed by the pharmaceutical industry are only modestly effective, and they have side effects that range from nausea and constipation to addiction and death.

What’s often overlooked is that the simple conversation between doctor and patient can be as potent an analgesic as many treatments we prescribe.

In 2014, researchers in Canada did an interesting study about the role of communication in the treatment of chronic back pain. Half the patients in the study received mild electrical stimulation from physical therapists, and half received sham stimulation (all the equipment is set up, but the electrical current is never activated). Sham treatment — placebo — worked reasonably well: These patients experienced a 25 percent reduction in their levels of pain. The patients who got the real stimulation did even better, though; their pain levels decreased by 46 percent. So the treatment itself does work.

Each of these groups was further divided in half. One half experienced only limited conversation from the physical therapist. With the other half, the therapists asked open-ended questions and listened attentively to the answers. They expressed empathy about the patients’ situation and offered words of encouragement about getting better.

Patients who underwent sham treatment but had therapists who actively communicated reported a 55 percent decrease in their pain. This is a finding that should give all medical professionals pause: Communication alone was more effective than treatment alone. The patients who got electrical stimulation from engaged physical therapists were the clear winners, with a 77 percent reduction in pain.

This type of study provides hard evidence for what shamans, witch doctors and assorted mystics have known for millenniums: A substantial portion of “healing” comes from the communication and connection with the patient.

Before we had treatments that could actually counteract the pathology of disease — antibiotics, chemotherapy, stents, organ transplants, transfusions — placebo was the mainstay of medical care, and in many cases it was remarkably effective.

A good example is patients suffering from vague diffuse pains with no discernible cause. Frequently my patients ask if a multivitamin will give them more energy. In the past I would say no, because there are no significant scientific studies to demonstrate this, and also because in the absence of a vitamin deficiency there’s not much for a basic multivitamin pill to do. Now I take a different approach. I say something along the lines of “Many of my patients find that they have more energy when they take a multivitamin.” I’m not lying, because many have indeed said so. Without fail, there are always a few patients who come back at the next visit and swear they feel much better.

There are some who argue that it is unethical to promote placebos to patients. But increasingly, many say it would be unethical not to give placebos a try in situations where patients are not getting relief from traditional means (and where it would not cause harm or replace a necessary treatment).

It’s clear that how doctors and nurses communicate their treatment can have profound effects on how patients experience the results of that treatment. Yet the conversation between doctors and patients is one of the least valued aspects of medical care…


9. Managing Migraine

Friedman BW. Ann Emerg Med. 2017;69(2):202-7.

Migraine is a recurrent headache disorder that afflicts 18% of US women and 9% of US men.1 It causes at least 1.2 million visits to US emergency department (EDs) annually; the actual number is probably substantially larger because many migraine patients are assigned nonspecific headache diagnostic codes.2 Migraine severity, as measured by the frequency with which it disrupts a patient’s life, ranges from minimal to severe. On one end of this spectrum are patients who have occasional headaches that are rapidly and effectively treated with over-the-counter therapies. On the other end are patients with chronic migraine. They have headache on more days than not and their work and social life is detrimentally affected.

An aura is one of several reversible neurologic phenomena that precede the headache and resolve completely. Most commonly, these are visual or sensory, although they may involve motor function or speech. Migraine patients also frequently report neurologic phenomena including dizziness, sensory disturbances, and visual symptoms during the acute attack. Because they occur during the headache, these latter symptoms are not typically referred to as aura. The migraine prodrome is a constellation of symptoms that precede the acute migraine attack by several days and include changes in mood, alertness, and appetite. Allodynia, an alteration of nociception that causes typically non-noxious sensory stimuli (such as brushing one’s hair or shaving one’s face) to be perceived as painful, develops as acute migraine duration increases. This is thought to indicate involvement of higher-order central nervous system sensory relay stations, notably, the thalamus.

Migraine was once believed to be a vascular headache. Advanced imaging studies do not support this description and indicate that migraine is a neurologic disorder involving dysfunctional nociceptive processing.3 Abnormally activated sensory pathways turn non-noxious stimuli into headache, photophobia, phonophobia, and osmophobia. Cortical spreading depression, a slow wave of brain depolarization, underlies migraine aura but has not been demonstrated clearly in migraine patients without aura….

The remainder of the review (full-text free): http://www.annemergmed.com/article/S0196-0644(16)30301-8/fulltext

10. Images in Clinical Practice

Gas in the Left Atrium and Ventricle

Eosinophilic Otitis Media

Calcific Pancreatitis Associated with Alcohol Use

Taenia solium

Swirl Sign — Intestinal Volvulus after Roux-en-Y Gastric Bypass

Intraabdominal vibrator due to a vaginal cuff dehiscence

Infant With Acute Respiratory Distress

EM-RAP Discussion: The Neonatal Airway and the Goldilocks Phenomenon

Young Male With Stab Wound

Woman With Altered Behavior

Young Man With Sudden Onset of Shortness of Breath

Man With Stridor

Elderly Female With Acute Vision Loss

Woman With Progressively Worsening Retiform Purpura

An Infant With Fever and Rash

Young Boy With Rash

11. Endotracheal Intubation Studies from JAMA

A. Negative Association between Tracheal Intubation During Adult In-Hospital Cardiac Arrest and Survival

Andersen LW, et al. JAMA.  2017;317(5):494-506.

Key Points 
Question  Is tracheal intubation during adult in-hospital cardiac arrest associated with survival?

Findings  In a study of 86 628 adults with in-hospital cardiac arrest using a propensity-matched cohort, tracheal intubation within the first 15 minutes was associated with a significantly lower likelihood of survival to hospital discharge compared with not being intubated (16.3% vs 19.4%, respectively).

Meaning  These findings do not support early tracheal intubation for adult in-hospital cardiac arrest.

Abstract 
Importance  Tracheal intubation is common during adult in-hospital cardiac arrest, but little is known about the association between tracheal intubation and survival in this setting.

Objective  To determine whether tracheal intubation during adult in-hospital cardiac arrest is associated with survival to hospital discharge.

Design, Setting, and Participants  Observational cohort study of adult patients who had an in-hospital cardiac arrest from January 2000 through December 2014 included in the Get With The Guidelines–Resuscitation registry, a US-based multicenter registry of in-hospital cardiac arrest. Patients who had an invasive airway in place at the time of cardiac arrest were excluded. Patients intubated at any given minute (from 0-15 minutes) were matched with patients at risk of being intubated within the same minute (ie, still receiving resuscitation) based on a time-dependent propensity score calculated from multiple patient, event, and hospital characteristics.

Exposure  Tracheal intubation during cardiac arrest.

Main Outcomes and Measures  The primary outcome was survival to hospital discharge. Secondary outcomes included return of spontaneous circulation (ROSC) and a good functional outcome. A cerebral performance category score of 1 (mild or no neurological deficit) or 2 (moderate cerebral disability) was considered a good functional outcome.

Results  The propensity-matched cohort was selected from 108 079 adult patients at 668 hospitals. The median age was 69 years (interquartile range, 58-79 years), 45 073 patients (42%) were female, and 24 256 patients (22.4%) survived to hospital discharge. Of 71 615 patients (66.3%) who were intubated within the first 15 minutes, 43 314 (60.5%) were matched to a patient not intubated in the same minute. Survival was lower among patients who were intubated compared with those not intubated: 7052 of 43 314 (16.3%) vs 8407 of 43 314 (19.4%), respectively (risk ratio [RR] = 0.84; 95% CI, 0.81-0.87; P  less than  .001). The proportion of patients with ROSC was lower among intubated patients than those not intubated: 25 022 of 43 311 (57.8%) vs 25 685 of 43 310 (59.3%), respectively (RR = 0.97; 95% CI, 0.96-0.99; P  less than  .001). Good functional outcome was also lower among intubated patients than those not intubated: 4439 of 41 868 (10.6%) vs 5672 of 41 733 (13.6%), respectively (RR = 0.78; 95% CI, 0.75-0.81; P  less than  .001). Although differences existed in prespecified subgroup analyses, intubation was not associated with improved outcomes in any subgroup.

Conclusions and Relevance  Among adult patients with in-hospital cardiac arrest, initiation of tracheal intubation within any given minute during the first 15 minutes of resuscitation, compared with no intubation during that minute, was associated with decreased survival to hospital discharge. Although the study design does not eliminate the potential for confounding by indication, these findings do not support early tracheal intubation for adult in-hospital cardiac arrest.

B. Video Laryngoscopy vs Direct Laryngoscopy on Successful First-Pass Orotracheal Intubation Among ICU Patients: A Randomized Clinical Trial

Lascarrou JB, et al. JAMA.  2017;317(5):483-493.

Key Points
Question: Should video laryngoscopy be used for orotracheal intubation in the intensive care unit (ICU) despite conflicting evidence that it improves the first-pass success rate?

Findings: Video laryngoscopy for orotracheal intubation in the ICU did not improve the first-pass success rate compared with conventional direct laryngoscopy (67.7% vs 70.3%, respectively).

Meaning: Video laryngoscopy did not improve the frequency of successful first-pass intubation in the ICU.

Abstract
Importance  In the intensive care unit (ICU), orotracheal intubation can be associated with increased risk of complications because the patient may be acutely unstable, requiring prompt intervention, often by a practitioner with nonexpert skills. Video laryngoscopy may decrease this risk by improving glottis visualization.

Objective  To determine whether video laryngoscopy increases the frequency of successful first-pass orotracheal intubation compared with direct laryngoscopy in ICU patients.

Design, Setting, and Participants  Randomized clinical trial of 371 adults requiring intubation while being treated at 7 ICUs in France between May 2015 and January 2016; there was 28 days of follow-up.

Interventions  Intubation using a video laryngoscope (n = 186) or direct laryngoscopy (n = 185). All patients received general anesthesia.

Main Outcomes and Measures  The primary outcome was the proportion of patients with successful first-pass intubation. The secondary outcomes included time to successful intubation and mild to moderate and severe life-threatening complications.

Results  Among 371 randomized patients (mean [SD] age, 62.8 [15.8] years; 136 [36.7%] women), 371 completed the trial. The proportion of patients with successful first-pass intubation did not differ significantly between the video laryngoscopy and direct laryngoscopy groups (67.7% vs 70.3%; absolute difference, −2.5% [95% CI, −11.9% to 6.9%]; P = .60). The proportion of first-attempt intubations performed by nonexperts (primarily residents, n = 290) did not differ between the groups (84.4% with video laryngoscopy vs 83.2% with direct laryngoscopy; absolute difference 1.2% [95% CI, −6.3% to 8.6%]; P = .76). The median time to successful intubation was 3 minutes (range, 2 to 4 minutes) for both video laryngoscopy and direct laryngoscopy (absolute difference, 0 [95% CI, 0 to 0]; P = .95). Video laryngoscopy was not associated with life-threatening complications (24/180 [13.3%] vs 17/179 [9.5%] for direct laryngoscopy; absolute difference, 3.8% [95% CI, −2.7% to 10.4%]; P = .25). In post hoc analysis, video laryngoscopy was associated with severe life-threatening complications (17/179 [9.5%] vs 5/179 [2.8%] for direct laryngoscopy; absolute difference, 6.7% [95% CI, 1.8% to 11.6%]; P = .01) but not with mild to moderate life-threatening complications (10/181 [5.4%] vs 14/181 [7.7%]; absolute difference, −2.3% [95% CI, −7.4% to 2.8%]; P = .37).

Conclusions and Relevance  Among patients in the ICU requiring intubation, video laryngoscopy compared with direct laryngoscopy did not improve first-pass orotracheal intubation rates and was associated with higher rates of severe life-threatening complications. Further studies are needed to assess the comparative effectiveness of these 2 strategies in different clinical settings and among operators with diverse skill levels.

12. (Minimal!?) Risk of Acute Kidney Injury after IV Contrast Media Administration.

Hinson JS, et al. Ann Emerg Med. 2017 Jan 19 [Epub ahead of print]

STUDY OBJECTIVE: The study objective was to determine whether intravenous contrast administration for computed tomography (CT) is independently associated with increased risk for acute kidney injury and adverse clinical outcomes.

METHODS: This single-center retrospective cohort analysis was performed in a large, urban, academic emergency department with an average census of 62,179 visits per year; 17,934 ED visits for patients who underwent contrast-enhanced, unenhanced, or no CT during a 5-year period (2009 to 2014) were included. The intervention was CT scan with or without intravenous contrast administration. The primary outcome was incidence of acute kidney injury. Secondary outcomes included new chronic kidney disease, dialysis, and renal transplantation at 6 months. Logistic regression modeling and between-groups odds ratios with and without propensity-score matching were used to test for an independent association between contrast administration and primary and secondary outcomes. Treatment decisions, including administration of contrast and intravenous fluids, were examined.

RESULTS: Rates of acute kidney injury were similar among all groups. Contrast administration was not associated with increased incidence of acute kidney injury (contrast-induced nephropathy criteria odds ratio=0.96, 95% confidence interval 0.85 to 1.08; and Acute Kidney Injury Network/Kidney Disease Improving Global Outcomes criteria odds ratio=1.00, 95% confidence interval 0.87 to 1.16). This was true in all subgroup analyses regardless of baseline renal function and whether comparisons were made directly or after propensity matching. Contrast administration was not associated with increased incidence of chronic kidney disease, dialysis, or renal transplant at 6 months. Clinicians were less likely to prescribe contrast to patients with decreased renal function and more likely to prescribe intravenous fluids if contrast was administered.

CONCLUSION: In the largest well-controlled study of acute kidney injury following contrast administration in the ED to date, intravenous contrast was not associated with an increased frequency of acute kidney injury.

Ryan Radecki comments

13. Responsible opioid use in emergency medicine.

A. Opioid-Prescribing Patterns of Emergency Physicians and Risk of Long-Term Use.

Barnett ML, et al. N Engl J Med. 2017 Feb 16;376(7):663-673. doi: 10.1056/NEJMsa1610524.

Background Increasing overuse of opioids in the United States may be driven in part by physician prescribing. However, the extent to which individual physicians vary in opioid prescribing and the implications of that variation for long-term opioid use and adverse outcomes in patients are unknown.

Methods We performed a retrospective analysis involving Medicare beneficiaries who had an index emergency department visit in the period from 2008 through 2011 and had not received prescriptions for opioids within 6 months before that visit. After identifying the emergency physicians within a hospital who cared for the patients, we categorized the physicians as being high-intensity or low-intensity opioid prescribers according to relative quartiles of prescribing rates within the same hospital. We compared rates of long-term opioid use, defined as 6 months of days supplied, in the 12 months after a visit to the emergency department among patients treated by high-intensity or low-intensity prescribers, with adjustment for patient characteristics.

Results Our sample consisted of 215,678 patients who received treatment from low-intensity prescribers and 161,951 patients who received treatment from high-intensity prescribers. Patient characteristics, including diagnoses in the emergency department, were similar in the two treatment groups. Within individual hospitals, rates of opioid prescribing varied widely between low-intensity and high-intensity prescribers (7.3% vs. 24.1%). Long-term opioid use was significantly higher among patients treated by high-intensity prescribers than among patients treated by low-intensity prescribers (adjusted odds ratio, 1.30; 95% confidence interval, 1.23 to 1.37; P less than 0.001); these findings were consistent across multiple sensitivity analyses.

Conclusions Wide variation in rates of opioid prescribing existed among physicians practicing within the same emergency department, and rates of long-term opioid use were increased among patients who had not previously received opioids and received treatment from high-intensity opioid prescribers.

B. Something for pain: Responsible opioid use in emergency medicine.

Strayer RJ, et al. Am J Emerg Med. 2017 Feb;35(2):337-341.

The United States is currently experiencing a public health crisis of opioid addiction, which has its genesis in an industry marketing effort that successfully encouraged clinicians to prescribe opioids liberally, and asserted the safety of prescribing opioids for chronic non-cancer pain, despite a preponderance of evidence demonstrating the risks of dependence and misuse. The resulting rise in opioid use has pushed drug overdose deaths in front of motor vehicle collisions to become the leading cause of accidental death in the country. Emergency providers frequently treat patients for complications of opioid abuse, and also manage patients with acute and chronic pain, for which opioids are routinely prescribed. Emergency providers are therefore well positioned to both prevent new cases of opioid misuse and initiate appropriate treatment of existing opioid addicts. In opioid-naive patients, this is accomplished by a careful consideration of the likelihood of benefit and harm of an opioid prescription for acute pain. If opioids are prescribed, the chance of harm is reduced by matching the number of pills prescribed to the expected duration of pain and selecting an opioid preparation with low abuse liability. Patients who present to acute care with exacerbations of chronic pain or painful conditions associated with opioid misuse are best managed by treating symptoms with opioid alternatives and encouraging treatment for opioid addiction.

C. Opioids and the Emergency Physician: Ducking Between Pendulum Swings (replay from August 2016 Lit Bits)

Yealy DM, et al. Ann Emerg Med. 2016;68:209-212.

In the late 1980s, undertreated pain became a focus of many clinicians, investigators, and industry. We coined a term, oligoanalgesia, to embolden earlier, titrated opioids and other analgesic use. Regulators mandated pain assessments and actions as a part of health care facility credentialing, and the prescribing of opioids increased yearly from 1990 to 2010.

We now sit amid another opioid epidemic with death and disability. Many know of the patterns of use and abuse near the turn of the 20th century when opium was a new agent; this triggered opioid regulations and control, notably, the US Harrison Narcotics Tax Act of 1914 criminalizing nonmedical use. Despite regulation of opioids, swings upward in medical and nonmedical use exist in recent modern times. For example, a 1959 Readers Digest  article noted a current opioid addiction crisis, reminding us that the cycles reappear often, with varying frequency, responses, and consequences.

The current opioid upswing is rooted in prescribed and illicit use, the latter including heroin (or other “street” opioids) and diversion of another’s or factitious gain of a prescribed opioid. The widespread increase in opioid use correlates with increased overdose deaths and deaths in all segments, including more affluent and suburban or rural groups, not just in an inner-city or isolated population. Although the peak of the current wave crested in approximately 2010, we retain a large group with this affliction. This triggered calls for regulation and control in many facets of opioid deployment, including emergency care, despite few data suggesting a clear link between emergency department (ED) opioid use and later harm. Some seek governmental ED dose restrictions, and others call for “an opioid-free ED.” The goal of these calls to action is good: to avoid opioid harm, notably, death. At the same time, we seek to provide relief of pain and wonder how our acute care episode affects the broader epidemic.

With all the dialogue, emergency physicians seek the answer to the question, “What am I supposed to do: skip the pain relief that an opioid delivers or risk addiction and overdose in some fraction of patients?”…

…More important in assessing the risk is knowing how often ED opioid analgesia turns patients into addicts. The data from Butler et al may help us estimate this risk. Our crude estimate indicates that 1 new heroin abuser might result from the administration of opioids to approximately 7,864 ED patients. An emergency physician administering opioids in the usual fashion could “prime” a new heroin addict once every 3 to 4 years of full-time practice. The number of total new heroin abusers attributable to first ED exposures appears to approximate 3,179 annually, a 1.1% contribution to the total US heroin addiction burden. Although the precise numbers we calculated could be inaccurate, we doubt these are differences of orders of magnitude, retaining the basic relationship. On top of this calculation, we posit that a much smaller unknown fraction will experience overdose or death, the group driving calls for action...

Chen et al report an opioid prescription analysis that includes 68% of the roughly 50 million US residents with Medicare insurance, finding that emergency physicians were ninth of the 25 specialties represented when the total number of opioid schedule II prescriptions were ranked. Indeed, physicians in the 8 higher-ranking specialties—most primary care or pain management—collectively wrote more than 25 times as many opioid prescriptions as emergency physicians did. In terms of opioid prescriptions per individual physician, emergency physicians ranked very low—18th of the 25 specialties. Although Medicare data sets do not include all US residents, it is unlikely that another analysis could assign a more prominent role in the opioid crisis to emergency physicians, tempering the need for aggressive ED reduction efforts. When the number and type of ED discharge opioid doses were examined, recent observations showed that prudence is common, with a regimen of usually 20 doses or fewer of a moderate-potency, short-acting agent. Not surprising is that efforts that have succeeded in cutting overdoses and deaths targeted the high-frequency prescribers and dispensers rather than ED prescribing.

Despite these observations, some EDs are implementing policies to decrease all opioid prescribing. One such approach withholds opioids for patients with migraine or preexisting back pain, those unable to obtain timely refills from their primary care provider, and those with any  previous opioid therapy from another physician. Many ED patients, particularly the poor, lack short-notice access to their primary care physicians. Are the sharp reductions in opioid use observed improving overall ED patient health or are they mostly symbolic? How many patients should have less relief to avoid the very small chance of downstream potential opioid abuse? We can only estimate, but the numbers are high, depending on the reduction in opioid prescribing sought.

The data from Butler et al  make it clear that we don’t need to take a blanket approach to refusing opioids because patients who linked an ED opioid exposure to heroin use are often men, young (average age of 18 years at first exposure), and with previous drug abuse, known commonalities for heroin abuse. The real value in meaningful ED opioid-prescribing reduction efforts is in focusing, especially on the addiction prone and those with opioid abuse or a diversion history. Said more clearly, we doubt providing short-course hydrocodone for Grandma’s flare of diverticulitis will drive her into the streets for heroin; writing an opioid prescription for those who “lost” their opioid is much more likely to trigger or enhance abuse and harm.

The zeal to avoid oligoanalgesia encouraged opioid prescribing for some patients when alternatives were a better choice; we wonder whether the backlash calling for “opioid-free EDs” or marked restrictions such as “only give 2 to 3 days” may create a different version of suboptimal care. We are uncertain what exactly “2 to 3 days” of analgesia entails, given the variability of need; can we accurately estimate how many doses are needed to ease pain with an acute fracture (opioids have a wide titration response range)  and  then ensure follow-up by the fourth day after the ED visit?

We think a better ED approach to limiting opioid addiction entails avoiding frequent ED opioid refills, placing prescription monitoring data at emergency providers’ fingertips during care, recognizing abuse patterns during an ED visit, and addressing and regularly considering alternative analgesic options—but not broad opioid restrictions. Let’s avoid a pendulum swing toward castigation or shackling of emergency providers seeking to responsibly relieve acute pain. ED opioids are a minor contributor to the current scourge of opioid addiction, with a likely far greater influence on this epidemic coming through the targeting of high-volume prescribers and illicit heroin sources.



14. ED Presentation of Patients with Spontaneous Coronary Artery Dissection.

Lindor RA, et al. J Emerg Med. 2017;52(3):286-91.  

BACKGROUND: Spontaneous coronary artery dissection (SCAD) is an infrequently recognized but potentially fatal cause of acute coronary syndrome (ACS) that disproportionately affects women. Little is currently known about how patients with SCAD initially present.

OBJECTIVES: We sought to describe patients who presented to the emergency department (ED) with symptoms of SCAD to improve providers' awareness and recognition of this condition.

PATIENTS AND METHODS: We performed a retrospective medical record review of all patients who presented to the ED of a single academic medical center from January 1, 2002 through October 31, 2015 and were subsequently diagnosed with SCAD by angiography. These patients were identified by International Classification of Diseases, Ninth Revision codes and a Boolean search of the diagnosis field of the medical record. Data regarding patients' presentations and course were abstracted by two independent reviewers.

RESULTS: We identified 20 episodes of SCAD involving 19 patients, all of whom were female. The majority of patients had 0-1 conventional cardiovascular disease risk factors. Most patients had chest pain (85%), initial electrocardiograms without evidence of ischemia (85%), and elevated initial troponin (72%). The most common diagnosis in providers' differential was acute coronary syndrome (ACS).

CONCLUSION: Patients with SCAD present with similar symptoms compared to patients with ACS caused by atherosclerotic disease, but have different risk profiles. Providers should consider SCAD in patients presenting with symptoms concerning for ACS, especially in younger female patients without traditional cardiovascular disease risk factors, as their risk may be significantly underestimated with commonly used ACS risk-stratifiers.

15. Lidocaine Pretreatment Reduces the Discomfort of Intranasal Midazolam Administration: A Randomized, Double-blind, Placebo-controlled Trial.

Smith D, et al. Acad Emerg Med. 2017 Feb;24(2):161-167.

OBJECTIVE: Intranasal (IN) midazolam is a commonly prescribed medication for pediatric sedation and anxiolysis. One of its most frequently encountered adverse effects is discomfort with administration. While it has been proposed that premedicating with lidocaine reduces this undesirable consequence, this combination has not been thoroughly researched. The objective of our study was to assess whether topical lidocaine lessens the discomfort associated with IN midazolam administration.

METHODS: This was a double-blind, randomized, placebo-controlled trial performed in an urban, academic pediatric emergency department. Children 6-12 years of age who were receiving IN midazolam for procedural sedation received either 4% lidocaine or 0.9% saline (placebo) via mucosal atomizer. Subjects were subsequently given IN midazolam in a similar fashion and then rated their discomfort using the Wong-Baker FACES Pain Rating Scale (WBS). The primary endpoint of WBS score was analyzed with a two-tailed Mann-Whitney U-test, with p less than 0.05 considered statistically significant.

RESULTS: Seventy-seven patients were enrolled over a consecutive 8-month period. One child was excluded from analysis due to a discrepancy in recording the drug identification number. Study groups were similar in regard to demographic information and indication for sedation. Subjects who received IN lidocaine reported less discomfort with IN midazolam administration (median WBS = 3, interquartile range [IQR] = 0-6) than those who received placebo (median WBS = 8, IQR = 2-9; p = 0.006).

CONCLUSIONS: Premedication with topical lidocaine reduces the discomfort associated with administration of IN midazolam

16. Best Clinical Practice: Controversies in TIA Evaluation and Disposition in the ED

Long B, et al. J Emerg Med. 2017;52(3):299-310.

BACKGROUND: Transient ischemic attack (TIA) affects over 200,000 patients annually in the United States, and it precedes approximately 14% to 23% of strokes. Patients are typically admitted for evaluation and management.

OBJECTIVE: This review investigates the controversies of emergency department (ED) evaluation of TIA, including imaging, clinical risk scores, rapid diagnostic protocols, and disposition.

DISCUSSION: TIA is a common condition, with over 200,000 patients affected annually, and is associated with risk of stroke. TIA is defined as a brief episode of neurologic dysfunction with no permanent infarction. A great deal of literature has evaluated the use of imaging, clinical risk scores, and diagnostic protocols in the evaluation of TIA. Head computed tomography noncontrast is not reliable to diagnose acute infarction. Magnetic resonance imaging with diffusion-weighted imaging displays greater diagnostic ability. Carotid imaging includes magnetic resonance angiography, computed tomography angiography, and Doppler with ultrasound. Risk scores that predict future stroke are not reliable when utilized alone. With imaging, including magnetic resonance imaging, patients with low-risk scores can be discharged. The use of ED diagnostic protocols and observation units can reduce length of stay while improving patient treatment and reducing stroke rate. An algorithm is provided for evaluation and disposition in the ED.

CONCLUSIONS: TIA is a condition with high risk for stroke. Imaging is often not reliable, nor is the use of risk scores alone. The American College of Emergency Physicians provides a Level B Recommendation for the use of rapid diagnostic protocols to determine patient short-term risk for stroke while avoiding the reliance on stratification instruments to discharge patients from the ED.

17. Impact of scribes on ED patient throughput one year after implementation.

Heaton HA, et al. Am J Emerg Med. 2017 Feb;35(2):311-314.

OBJECTIVES: Assess the impact of scribes on an academic emergency department's (ED) throughput one year after implementation.

METHODS: A prospective cohort design compared throughput metrics of patients managed when scribes were and were not a part of the treatment team during pre-defined study hours in a tertiary academic ED with both an adult and pediatric ED. An alternating-day pattern one year following scribe implementation ensured balance between the scribe and non-scribe groups in time of day, day of week, and patient complexity.

RESULTS: Adult: Overall length of stay (LOS) was essentially the same in both groups (214 vs. 215min, p=0.34). In area A where staffing includes an attending and residents, scribes made a significant impact in treatment room time in the afternoon (190 vs 179min, p=0.021) with an increase in patients seen per hour on scribed days (2.00 vs. 2.13). There was no statistically significant changes in throughput metrics in area B staffed by an attending and a nurse practitioner/physician assistant, however scribed days did average more patients per hour (2.01 vs. 2.14). Pediatric: All throughput measurements were significantly longer when the treatment team had a scribe; however, patients per hour increased from 2.33 to 2.49 on scribed days.

CONCLUSIONS: Overall patient throughput was not enhanced by scribes. Certain areas and staffing combinations yielded improvements in treatment room and door to provider time, however, scribes appear to have enabled attending physicians to see more patients per hour. This effect varied across treatment areas and times of day.

18. Leukoaraiosis predicts ICH after acute stroke thrombolysis

Kongbunkiat K, et al. Leukoaraiosis, ICH, and functional outcome after acute stroke thrombolysis Neurology. 2017 Feb 14;88(7):638-645.

OBJECTIVE: To perform a systematic review and pooled meta-analysis of published studies to assess whether the presence of leukoaraiosis on neuroimaging before treatment with thrombolysis (IV or intra-arterial) is associated with an increased risk of symptomatic intracerebral hemorrhage (sICH) or poor functional outcome.

METHODS: We included studies of patients with acute ischemic stroke, treated with IV or intra-arterial thrombolysis, which assessed functional outcome (3-month modified Rankin Scale [mRS]) or sICH in relation to leukoaraiosis on pretreatment neuroimaging (CT or MRI). We used random-effects models to calculate pooled relative risks (RR) of sICH and poor functional outcome (mRS ≥ 2) for any vs no leukoaraiosis (using any rating scale) and for no to mild vs moderate to severe leukoaraiosis (using the Van Swieten or Fazekas Schmidt scale).

RESULTS: We identified 15 studies (total n = 6,967). For sICH outcome, the RR was 1.65 (n = 5,551; 95% confidence interval [CI] 1.26-2.16, p = 0.001) with an absolute risk (AR) increase of 2.5% for any leukoaraiosis vs none. The RR was 2.4 (n = 4,192; 95% CI 1.83-3.14, p = 0.001) with an AR increase of 6.2% for moderate to severe vs no to mild leukoaraiosis. For poor functional outcome; the RR was 1.30 (n = 3,401; 95% CI 1.19-1.42, p = 0.001) with an AR increase of 15.4% for any leukoaraiosis vs none. The RR was 1.31 (n = 3,659; 95% CI 1.22-1.42, p = 0.001) with an AR increase of 17.5% for moderate to severe vs no to mild leukoaraiosis. No statistical heterogeneity was noted for any of the analyses.

CONCLUSIONS: Leukoaraiosis presence and severity are consistently associated with an increased risk of sICH and poor functional outcome after IV or intra-arterial thrombolysis for acute ischemic stroke.


19. Associations of ED LOS with Publicly Reported Quality-of-care Measures.

Chang AM, et al. Acad Emerg Med. 2017 Feb;24(2):246-250.

OBJECTIVE: The Institute of Medicine identified emergency department (ED) crowding as a critical threat to patient safety. We assess the association between changes in publicly reported ED length of stay (LOS) and changes in quality-of-care measures in a national cohort of hospitals.

METHODS: Longitudinal analysis of 2012 and 2013 data from the American Hospital Association (AHA) Survey, Center for Medicare and Medicaid Services (CMS) Cost Reports, and CMS Hospital Compare. We included hospitals reporting Hospital Compare timeliness measure of LOS for admitted patients. We used AHA and CMS data to incorporate hospital predictors of interest. We used the method of first differences to test for relationships in the change over time between timeliness measures and six hospital-level measures.

RESULTS: The cohort consisted of 2,619 hospitals. Each additional hour of ED LOS was associated with a 0.7% decrease in proportion of patients giving a top satisfaction rating, a 0.7% decrease in proportion of patients who would "definitely recommend" the hospital, and a 6-minute increase in time to pain management for long bone fracture (p less than 0.01 for all). A 1-hour increase in ED LOS is associated with a 44% increase in the odds of having an increase in left without being seen (95% confidence interval = 25% to 68%). ED LOS was not associated with hospital readmissions (p = 0.14) or time to percutaneous coronary intervention (p = 0.14).

CONCLUSION: In this longitudinal study of hospitals across the United States, improvements in ED timeliness measures are associated with improvements in the patient experience.

20. PE in Patients Hospitalized for Syncope: the Controversial NEJM Study from Italy

The responses from readers in the letters to the editor: N Engl J Med 2017; 376:494-497.

Letters and the authors’ reply here: http://www.nejm.org/doi/full/10.1056/NEJMc1615913


21. No asthma found in one third of adults previously diagnosed with it

Researchers blame the high number on misdiagnosis and spontaneous remission

Reevaluation of Diagnosis in Adults With Physician-Diagnosed Asthma

Aaron SD, et al. JAMA.  2017;317(3):269-279.

Importance  Although asthma is a chronic disease, the expected rate of spontaneous remissions of adult asthma and the stability of diagnosis are unknown.

Objective  To determine whether a diagnosis of current asthma could be ruled out and asthma medications safely stopped in randomly selected adults with physician-diagnosed asthma.

Design, Setting, and Participants  A prospective, multicenter cohort study was conducted in 10 Canadian cities from January 2012 to February 2016. Random digit dialing was used to recruit adult participants who reported a history of physician-diagnosed asthma established within the past 5 years. Participants using long-term oral steroids and participants unable to be tested using spirometry were excluded. Information from the diagnosing physician was obtained to determine how the diagnosis of asthma was originally made in the community. Of 1026 potential participants who fulfilled eligibility criteria during telephone screening, 701 (68.3%) agreed to enter into the study. All participants were assessed with home peak flow and symptom monitoring, spirometry, and serial bronchial challenge tests, and those participants using daily asthma medications had their medications gradually tapered off over 4 study visits. Participants in whom a diagnosis of current asthma was ultimately ruled out were followed up clinically with repeated bronchial challenge tests over 1 year.

Exposure  Physician-diagnosed asthma established within the past 5 years.

Main Outcomes and Measures  The primary outcome was the proportion of participants in whom a diagnosis of current asthma was ruled out, defined as participants who exhibited no evidence of acute worsening of asthma symptoms, reversible airflow obstruction, or bronchial hyperresponsiveness after having all asthma medications tapered off and after a study pulmonologist established an alternative diagnosis. Secondary outcomes included the proportion with asthma ruled out after 12 months and the proportion who underwent an appropriate initial diagnostic workup for asthma in the community.

Results  Of 701 participants (mean [SD] age, 51 [16] years; 467 women [67%]), 613 completed the study and could be conclusively evaluated for a diagnosis of current asthma. Current asthma was ruled out in 203 of 613 study participants (33.1%; 95% CI, 29.4%-36.8%). Twelve participants (2.0%) were found to have serious cardiorespiratory conditions that had been previously misdiagnosed as asthma in the community. After an additional 12 months of follow-up, 181 participants (29.5%; 95% CI, 25.9%-33.1%) continued to exhibit no clinical or laboratory evidence of asthma. Participants in whom current asthma was ruled out, compared with those in whom it was confirmed, were less likely to have undergone testing for airflow limitation in the community at the time of initial diagnosis (43.8% vs 55.6%, respectively; absolute difference, 11.8%; 95% CI, 2.1%-21.5%).

Conclusions and Relevance  Among adults with physician-diagnosed asthma, a current diagnosis of asthma could not be established in 33.1% who were not using daily asthma medications or had medications weaned. In patients such as these, reassessing the asthma diagnosis may be warranted.

22. Micro Bits

A. The Flexibility of Truth in the Information Age: Research on Bovine Excrement Detection Wins Ig Nobel Prize

Berger E. Ann Emerg Med 2017;69(2):A17-19.

Before discussing the research itself and its implications, it’s worth relating a few words about the apparent demise of factual information in the United States. A Chicago Tribune columnist, Rex Huppke, MA, wrote a tongue-in-cheek “obituary” for Facts 4 years ago, saying Facts had lived a long life from 360 bc to ad 2012…


B. I’m not a doctor, but I play one on my CV

By Adam RubenJan. Science. Jan 18, 2017.


C. Do vitamin D supplements help prevent respiratory tract infections?

No.


D. Financial ties of principal investigators and randomized controlled trial outcomes: cross sectional study

Ahn R, et al. BMJ 2017;356:i6770.


E. 350 Organizations Send Letter to Trump Touting Vaccines' Safety, Efficacy

Feb. 13, 2017 — The AAFP joined a broad coalition of more than 350 organizations that sent a letter to President Donald Trump emphasizing the safety and efficacy of vaccines.


Related: Most US adults favor required childhood MMR vaccinations

Eighty-two percent of 1,549 adults in the US said they support mandatory measles, mumps and rubella vaccinations for children in public schools and 88% said that MMR vaccines were more beneficial than harmful, according to a Pew Research Center survey. Researchers also found 73% said that decisions on childhood vaccine policies should be primarily influenced by medical scientists, compared with 25% of those who cited elected officials.


F. Top Preventive Services Recommendations Ranked by Relative Efficacy: Pediatric Immunizations, Smoking Cessation/Prevention, Alcohol Misuse, and Obesity Reduction

RESULTS The 3 highest-ranking services, each with a total score of 10, are immunizing children, counseling to prevent tobacco initiation among youth, and tobacco-use screening and brief intervention to encourage cessation among adults. Greatest population health improvement could be obtained from increasing utilization of clinical preventive services that address tobacco use, obesity-related behaviors, and alcohol misuse, as well as colorectal cancer screening and influenza vaccinations.


G. 'Overwhelming Efficacy': Rivaroxaban cut CV events in coronary and peripheral disease patients

he first large trial of a newer oral anticoagulant (NOAC) for prevention among coronary artery disease (CAD) and peripheral artery disease (PAD) -- the COMPASS study -- has been stopped early for "overwhelming efficacy," Bayer AG and Janssen, manufacturers of rivaroxaban (Xarelto), announced.

The phase III trial randomized 27,402 CAD and PAD patients to receive either rivaroxaban 2.5 mg twice daily in addition to aspirin 100 mg once daily, rivaroxaban 5 mg twice daily alone, or aspirin 100 mg once daily alone.

The trial was stopped a year ahead of its scheduled completion date following a recommendation by the Data Monitoring Committee. That committee said the trial reached its prespecified criteria for superiority on the primary endpoint of first occurrence of either MI, stroke, or cardiovascular death.

In an open-label extension trial, patients enrolled in the study will be offered rivaroxaban "owing to the magnitude of effect and the confirmation of the existing safety profile of rivaroxaban," the companies announced.

Full results of the study will be presented later this year at a major medical meeting.

The study: Rivaroxaban for the Prevention of Major Cardiovascular Events in Coronary or Peripheral Artery Disease (COMPASS): https://clinicaltrials.gov/ct2/show/NCT01776424

H. NSAIDs during respiratory infections increases AMI risk

A study in the Journal of Infectious Diseases found patients who took nonsteroidal anti-inflammatory drugs during a respiratory infection had a 3.4 times higher risk of heart attack, compared with a 2.7 times increased risk in patients who did not use the drugs during a respiratory infection. The findings, based on data for almost 10,000 patients hospitalized for a heart attack, also showed a 7.2-fold increased heart attack risk among patients who received the drugs intravenously while hospitalized.


I. Diabetes is third leading cause of death in US, study finds

Diabetes accounts for 11.5% and 11.7% of deaths based on self-reported diabetes in the National Health Interview Survey and the National Health and Nutrition Examination Survey, respectively, and 11.8% based on HbA1C levels in NHANES, making it the third leading cause of death in the US, researchers reported in PLOS ONE. Researchers found that the highest proportion of deaths linked to diabetes was among obese people at 19.4%.

"Responsibility for approximately 12% of deaths would make diabetes the third leading cause of death in the United States in 2010, after diseases of the heart and malignant neoplasms and ahead of chronic lower respiratory diseases and cerebrovascular diseases," Drs Stokes and Preston write.


J. AHA: Meal planning, eating breakfast may lower CVD risk

A scientific statement from the American Heart Association said meal planning and eating breakfast daily may help reduce cardiovascular disease risks. The statement, published in the journal Circulation, also said eating more early in the day and less at night may reduce the likelihood of cardiac and blood vessel diseases.


K. Prostate Cancer Screening Questions to Think about for Men Aged 50-74

An education video for pts: https://www.youtube.com/watch?v=v5z2Go4ZpO4

Video Decision Aids Help Patients Follow USPSTF Recommendations: http://www.aafp.org/news/health-of-the-public/20170130annfammedvideo.html

L. Sepsis accounts for 12.2% of hospital readmissions

A study in the Journal of the American Medical Association showed sepsis accounted for 12.2% of hospital readmissions, compared with 6.7% for heart failure, 5% for pneumonia, 4.6% for chronic obstructive pulmonary disease and 1.3% for heart attack, which are conditions included in a CMS readmission penalty program. Data showed sepsis also had higher readmission-related costs compared with the other diagnoses.



M. Effect of a Scalp Cooling Device on Alopecia in Women Undergoing Chemotherapy for Breast Cancer: The SCALP Randomized Clinical Trial

Question  What is the effectiveness of a scalp cooling device in preventing alopecia in women with breast cancer undergoing neoadjuvant or adjuvant chemotherapy?

Findings  In a randomized clinical trial of 182 women with breast cancer receiving chemotherapy with a taxane, anthracycline, or both, those who underwent scalp cooling were significantly more likely to have less than 50% hair loss compared with no scalp cooling (50.5% vs 0%). The trial was stopped early for superiority, and there was no effect on measures of quality of life.

Meaning  This scalp cooling system was more likely to prevent alopecia than no treatment, and further research is needed to assess longer-term efficacy and adverse effects.